GENERAL PRACTITIONER EXAM
Question Summary
0 of 100 questions completed
Questions:
- 1
- 2
- 3
- 4
- 5
- 6
- 7
- 8
- 9
- 10
- 11
- 12
- 13
- 14
- 15
- 16
- 17
- 18
- 19
- 20
- 21
- 22
- 23
- 24
- 25
- 26
- 27
- 28
- 29
- 30
- 31
- 32
- 33
- 34
- 35
- 36
- 37
- 38
- 39
- 40
- 41
- 42
- 43
- 44
- 45
- 46
- 47
- 48
- 49
- 50
- 51
- 52
- 53
- 54
- 55
- 56
- 57
- 58
- 59
- 60
- 61
- 62
- 63
- 64
- 65
- 66
- 67
- 68
- 69
- 70
- 71
- 72
- 73
- 74
- 75
- 76
- 77
- 78
- 79
- 80
- 81
- 82
- 83
- 84
- 85
- 86
- 87
- 88
- 89
- 90
- 91
- 92
- 93
- 94
- 95
- 96
- 97
- 98
- 99
- 100
Information
Hi, Welcome to GENERAL PRACTITIONER EXAM
You have already completed the Exam before. Hence you can not start it again.
Exam is loading...
You must sign in or sign up to start the Exam.
You have to finish following Exam, to start this Exam:
Results
0 of 100 questions answered correctly
Your time:
Time has elapsed
You have reached 0 of 0 points, (0)
Average score |
|
Your score |
|
Categories
- Not categorized 0%
Pos. | Name | Entered on | Points | Result |
---|---|---|---|---|
Table is loading | ||||
No data available | ||||
- 1
- 2
- 3
- 4
- 5
- 6
- 7
- 8
- 9
- 10
- 11
- 12
- 13
- 14
- 15
- 16
- 17
- 18
- 19
- 20
- 21
- 22
- 23
- 24
- 25
- 26
- 27
- 28
- 29
- 30
- 31
- 32
- 33
- 34
- 35
- 36
- 37
- 38
- 39
- 40
- 41
- 42
- 43
- 44
- 45
- 46
- 47
- 48
- 49
- 50
- 51
- 52
- 53
- 54
- 55
- 56
- 57
- 58
- 59
- 60
- 61
- 62
- 63
- 64
- 65
- 66
- 67
- 68
- 69
- 70
- 71
- 72
- 73
- 74
- 75
- 76
- 77
- 78
- 79
- 80
- 81
- 82
- 83
- 84
- 85
- 86
- 87
- 88
- 89
- 90
- 91
- 92
- 93
- 94
- 95
- 96
- 97
- 98
- 99
- 100
- Unanswered
- Answered
- Review
-
Question 1 of 100
1. Question
1 pointsA 74-year-old male with type 2 diabetes requires controlling his diabetes. He also has heart failure for which he is on a diuretic and an ACE inhibitor. Which of the following hypoglycemic agents is contraindicated in this patient?
Correct
Incorrect
Explanation:
Rosiglitazone can result in fluid retention of unknown etiology which may cause a mild dilutional anaemia (haemoglobin typically falls by 1 to 2 g/dL) and ankle edema.
It is contraindicated in hepatic dysfunction and congestive heart failure.
Sulphonylureas are contraindicated in renal failure. All should be stooped and insulin started if serum creatinine exceeds 250µmol/L.
The recent United Kingdom Prospective Diabetes Study found no evidence events that patients treated with sulphonylureas suffered cardiovascular events more often than those treated with insulin.
Contraindications of metformin include all the major organ failures:
– Renal
– Hepatic
– Cardiac and
– Respiratory
It should not be used when serum creatinine concentration exceeds 150µmol/L.
Side-effects of acarbose are due to carbohydrate malabsorption (flatus, abdominal bloating and gassy diarrhea). -
Question 2 of 100
2. Question
1 pointsA 57 year old woman is scheduled for an abdominal operation. She has hypothyroidism that is controlled with thyroid replacement medication and will be unable to eat or drink for 4 days following the procedure. She is concerned about receiving her thyroid medication. What should be advised to her?
Correct
Incorrect
Explanation:
Thyroxine is the hormone of choice for replacement therapy. It has a half life of seven days, and any alteration in dose is not reflected for four to six weeks. Therefore, it is very unlikely that she will develop signs and symptoms of hypothyroidism. She will not be given the medication either through the nasogastric tube or intravenously, nor does she require a preoperative loading dose.
-
Question 3 of 100
3. Question
1 pointsWhich one of the following approaches to prednisone withdrawal in patients who have received chronic daily therapy is best?
Correct
Incorrect
Explanation:
Patients who have been on chronic daily oral therapy with prednisone must be tapered slowly, often over many months. Abrupt cessation or very rapid tapering over a short period of time can result in Addisonian crisis. If alternate day use is desired, it must be achieved slowly. An agent such as methotrexate is not substituted for prednisone. It is sometimes given to appropriate patients, such as those with rheumatoid arthritis or lupus, as a remittive agent that has so-called “steroid-sparing” attributes, but it is not used in general as a single-drug therapy, substituting for prednisone.
-
Question 4 of 100
4. Question
1 pointsWhich one of the following individuals would be the best candidate for use of an insulin pump?
Correct
Incorrect
Explanation:
A patient who is compliant with his diet and his insulin, tests his blood as directed, and still has glucose levels that are out of control is the best candidate for implantation of an insulin pump that would provide continuous administration of insulin. Use of the pump is neither necessary nor medically appropriate in any of the other types of patients.
-
Question 5 of 100
5. Question
1 pointsNephrolithiasis is a common mode of presentation of which of the following disorder?
Correct
Incorrect
Explanation:
Renal stones due to hypercalciuria are a common presentation of primary hyperparathyroidism. The other conditions are not associated with the nephrolithiasis.
-
Question 6 of 100
6. Question
1 pointsA condition that is NOT associated with insulin resistance is which of the following?
Correct
Incorrect
Explanation:
Insulinomas are not associated with insulin resistance. All of the other conditions are associated with such resistance.
-
Question 7 of 100
7. Question
1 pointsWhat are the classic follicular stimulating hormone(FSH) and luteinizing hormone(LH) levels in Turner´s syndrome?
Correct
Incorrect
Explanation:
Turner´s syndrome (only one X sex chromosome 45XO) is associated with ovarian failure and lack of estrogen production. Thus, FSH and LH are not inhibited and, consequently, the levels are both elevated.
-
Question 8 of 100
8. Question
1 pointsCalcitonin is synthesized by which of the following cell types?
Correct
Incorrect
Explanation:
Calcitonin is synthesized by the C cells of the thyroid, not by any of the other cells listed. Sertoli cells secrete androgen-binding protein, inhibin, and MIS (mullerian inhibiting substance). Leydig cells secrete testosterone. Langerhans cells secrete insulin. Pituitary chromophobes are resting (nonsecretory) cells.
-
Question 9 of 100
9. Question
1 pointsWhich one of the following is FALSE regarding hyperparathyroidism?
Correct
Incorrect
Explanation:
Hypercalcemia may occur in persons taking lithium or thiazides due to increased parathyroid secretion. The primary disturbance in hyperparathyroidism is an inappropriate secretion of parathyroid hormone (PTH) for the level of serum calcium. The incidence has significantly increased as a result of routine screening with multichannel chemistry profiles. Subperiosteal bone resorption occurs at very high levels of PTH. Eventually, marrow fibrosis associated with cystic, reparative bone lesions develop. This classic bone disease is known as osteitis fibrosa cystica. Nephrolithiasis, composed of calcium oxalate or calcium phosphate, commonly occurs in persons with hyperparathyroidism and may result from hypercalcuria.
-
Question 10 of 100
10. Question
1 pointsA black obese female of age 52 years presents with new onset polyuria, polydipsia, weakness, and fatigue. Her fasting glucose level is 301 mg/dl. A family history of diabetes mellitus is present. Which of the following statement is FALSE?
Correct
Incorrect
Explanation:
The goal in diabetes control should be to keep the hemoglobin A1c level below 8%. By the time one presents with symptoms of type II diabetes, some end organ damage has been done. The diabetes control and complications trail has been one of the largest diabetes trails in the nation. It showed that tight control of glucose will result in slow progression of retinopathy and nephropathy. Diabetes is a strong risk factor for myocardial infarction, stroke, peripheral vascular disease, and generalized atherosclerosis. Three to five percent of the United States population has diabetes. Sixteen million people (diagnosed and undiagnosed) have diabetes. Diabetes is two to five times more common in minorities. Sixty to seventy percent of all diabetics are non insulin dependent and 10% are juvenile onset diabetics.
-
Question 11 of 100
11. Question
1 pointsAn infertile female aged 22 years who has oligomenorrhea and galactorrhea with a prolactin level of 1,157 ng/ml and low estrogen level is diagnosed as having hyperprolactinemia. Which of the following is NOT a cause of hyperprolactinemia?
Correct
Incorrect
Explanation:
Bromocriptine is a dopamine agonist. Side effects include nausea, vomiting, and mood changes. It inhibits prolactin secretion in all forms of prolactinemia. Galactorrhea is eliminated and cyclic menses and fertility may return without complete normalization of prolactin levels. Microadenoma and macroadenoma of the pituitary gland are the most common causes of hyperprolactinemia. Surgical resection results in a cure. Relapse of the tumor may occur. Empty sella syndrome is diagnosed by CT or MRI of the brain.
-
Question 12 of 100
12. Question
1 pointsThyroid carcinoma that is associated with an elevation of serum calcitonin levels is which one of the following?
Correct
Incorrect
Explanation:
Medullary thyroid carcinoma is a neuroendocrine mass arising from thyroid cells that secrete calcitonin, the tumor´s biochemical marker. Eighty percent are sporadic and 20% are familial bilateral tumors inherited as a component of multiple endocrine neoplasia (MEN syndromes-IIA and IIB). Thyroid lymphoma is rare and occurs in patients with long standing Hashimoto´s lymphocytic thyroiditis. Papillary thyroid cancer is the most common thyroid cancer and is more common in females. Thyroid malignancies arising after exposure to ionizing radiation are usually papillary carcinomas. Follicular thyroid cancers account for about 5-10% of all thyroid malignancies. It is more commonly seen in women, and usually presents as a single discrete encapsulated tumor surrounded by normal thyroid tissue. Anaplastic thyroid cancers occur mainly in the elderly, affect men and women equally and confer a poor prognosis, rapidly metastasizing to distant sites such as bone, lungs, and brain.Medullary thyroid carcinoma is a neuroendocrine mass arising from thyroid cells that secrete calcitonin, the tumor´s biochemical marker. Eighty percent are sporadic and 20% are familial bilateral tumors inherited as a component of multiple endocrine neoplasia (MEN syndromes-IIA and IIB). Thyroid lymphoma is rare and occurs in patients with long standing Hashimoto´s lymphocytic thyroiditis. Papillary thyroid cancer is the most common thyroid cancer and is more common in females. Thyroid malignancies arising after exposure to ionizing radiation are usually papillary carcinomas. Follicular thyroid cancers account for about 5-10% of all thyroid malignancies. It is more commonly seen in women, and usually presents as a single discrete encapsulated tumor surrounded by normal thyroid tissue. Anaplastic thyroid cancers occur mainly in the elderly, affect men and women equally and confer a poor prognosis, rapidly metastasizing to distant sites such as bone, lungs, and brain.
-
Question 13 of 100
13. Question
1 pointsA 36 year old woman with a painless palpable neck mass undergoes hemithyroidectomy. The histopathology shows a single large nodule occupying one pole, which on frozen section shows benign colloid follicles. The nodule is delimited by bands of fibrous tissue. The most likely diagnosis is
Correct
Incorrect
Explanation:
Thyroid (follicular) adenoma is a common benign lesion that is often surgically resected because of concern that it may be cancer. The lesion consists of a single large nodule of thyroid tissue (up to 5 cm diameter) the histology of which is often similar to that of the normal thyroid gland. Patients are generally euthyroid. Diffuse nontoxic goiter and multinodular goiter are stages of the same process, with multinodular goiter producing a large mass with a more nodular texture.
Subacute thyroiditis causes painful thyroid enlargement. A thyroid cyst is a common benign cystic space that develops in some thyroids. -
Question 14 of 100
14. Question
1 pointsA 46 year old man presents because of a bulge in his neck. Patient´s thyroid gland is enlarged on examination. Needle aspiration of the thyroid demonstrates cell clusters that are suspicious for papillary carcinoma of the thyroid. Finding that would strongly support this diagnosis is
Correct
Incorrect
Explanation:
The correct answer is C. Features of papillary carcinoma of the thyroid on aspiration include papillary clusters, “Orphan Annie” nuclei, and psammoma bodies. Psammoma bodies are laminated, concentric, calcific spherules seen most frequently in papillary adenocarcinoma of the thyroid, serous papillary cystadenocarcinoma of the ovary, meningioma, and malignant mesothelioma. Anitschkow cells are activated histiocytes found in rheumatic heart disease. Auer rods are cytoplasmic inclusions found in promyelocytic leukemia cells. Reed Sternberg cells are typically binucleate (or contain bibbed nuclei) with prominent “owl´s-eye” nucleoli cells, found in Hodgkin disease. Roth´s spots are pale retinal spots surrounded by hemorrhage, seen in endocarditis.
-
Question 15 of 100
15. Question
1 pointsA 23 year old female has never menstruated. Her pubic and axillary hairs are absent. Her breasts are small and a uterus is not palpable. Karyotype is 46, XY. DNA analysis shows point mutation in one of the codons in the androgen receptor gene. Gonads are removed after exploratory surgery. Histologic exam of the gonads reveal the presence of seminiferous tubules with normal Leydig cells. Prior to surgery (when compared to that of a normal woman) labs would have shown which of the following?
Correct
Incorrect
Explanation:
DNA analysis shows that this patient is a genotypic male with an abnormality of the androgen receptor molecule. The androgen receptor molecule normally combines with testosterone inside the cell, and the complex binds with chromatin, resulting in the synthesis of messenger RNA. Because the target tissues contain detective androgen receptor protein, they are unable to respond to testosterone. Androgen insensitivity during the fetal period means that male sexual development is impaired and female external genitalia are formed. Because the androgen receptor in hypothalamic and pituitary tissue is also defective, normal testosterone suppression of pituitary gonadotrophs, such as luteinizing hormone (LH), is absent. As a result, serum levels of gonadotropins are elevated resulting in increased production of testosterone by Leydig cells. High testosterone and low LH would not be expected in the androgen insensitivity syndrome. Cells of the pituitary and hypothalamus also contain the androgen receptor protein. Feedback suppression of LH secretion occurs when the molecular complex is present in the hypothalamus and pituitary. Testosterone is low and LH is high in primary hypogonadism due to detective gonadal steroid synthesis. The reduced testosterone results in feedback elevation of LH secretion by the pituitary. Low LH insulting in depressed levels of testosterone secretion is characteristic of a detect in hypothalamic pituitary function (hypogonadotrophic hypogonadism).
-
Question 16 of 100
16. Question
1 pointsA 34 year old otherwise healthy woman has episodic sweats associated with hunger. She gained some weight recently. Labs reveal normal urea and electrolytes, LFTs and full blood count. Overnight fasting plasma glucose is 3.8 mmol/L (3.0-6.0). The most appropriate investigation for this patient is
Correct
Incorrect
Explanation:
This patient presents with features suggestive of spontaneous hypoglycaemia often due to an insulinoma. She requires confirmation of the suspected diagnosis and this should be undertaken with a 72 hr fast. If the patient develops symptoms then a plasma glucose is measured and if low, insulin and c-peptide is then collected and the fast terminated. We have been provided with fasting plasma glucose of this patient which is normal. Measuring insulin and C-peptides with this normal glucose would provide no meaningful information. First we have to see whether she actually becomes hypoglycaemic.
-
Question 17 of 100
17. Question
1 pointsA type 1 diabetic aged 17 years presents for annual review. He takes three times daily short acting insulin with evening dose long acting insulin. His glycaemic control is good as reflected by an HbA1c of 6.5% (3.8-6.4). He seeks advice regarding his ability to pursue a future career. Which occupation would he be able to pursue?
Correct
Incorrect
Explanation:
Careers opportunities are affected by insulin use in Diabetes Mellitus and it is important to know these restrictions in order that you can provide appropriate advice to your patient. Any employment in the armed forces, fire service or police force is not permissible unless already a member of the armed forces. Offshore work again is not an option.
-
Question 18 of 100
18. Question
1 pointsA diagnosis of diabetes mellitus is being considered in 32-year-old woman who is 16 weeks pregnant. Her BMI was 22 kg/m2 (18 -25). A 75g oral glucose tolerance test revealed:
Time Plasma glucose concentration 0 hr 6.0 mmol/l (3.0-6.0) 2hr 12.5 mmol/l (<11.1) What is the next most appropriate step in her management?
Correct
Incorrect
Explanation:
The result confirms a diagnosis of gestational diabetes mellitus with the 2hr OGTT result above 11.1mmol/1. To minimize the fetal consequences of GDM (macrosomia, fetal malformations, still birth, IUGR etc), the patient´s glycaemia should be strictly controlled with insulin. A low calorie diet is inappropriate and neither metformin nor glipizide are licenced for use in pregnancy. There is no point in repeating the OGTT in 4 weeks as control is required NOW.
-
Question 19 of 100
19. Question
1 pointsA 54 year old woman presents with episodic sweats and tremors which are relieved by glucose. She has gained approximately 6 kg recently and drinks approximately 10 units of alcohol weekly. Full blood count, urea and electrolytes are normal and fasting plasma glucose concentration of 4mmol/L (3.0-6.0). The most appropriate investigation for this patient is which one of the following?
Correct
Incorrect
Explanation:
This patient describes symptoms suggestive of hypoglycaemia which a relieved by carbohydrate. The likely cause is an insulinoma which is producing the weight gain. The standard method for achieving a diagnosis is during a 72 hour fast by demonstration of inappropriately high insulin and C peptide during spontaneous hypoglycaemia. Measurement of C-peptide is useful for excluding factitious hypoglycaemia from self injection of insulin. Insulin preparations contain no C-peptide.
-
Question 20 of 100
20. Question
1 pointsA 57 year old presented after an episode of collapse at home. He had been feeling increasingly tired for the last 2 months and also has loss of libido. He underwent a trans-sphenoidal surgery 2 years ago, followed by external beam radiation for a non functional pituitary adenoma. His BP measured 104/66 mmHg in the lying position, dropping to 80/40 mmHg on standing. Lab data revealed hyponatremia. Plasma LH is 0.3 U/L (1-10) and TSH is 0.1 mU/L (0.4-5). The next appropriate investigation is
Correct
Incorrect
Explanation:
The most appropriate investigation in this patient who has biochemistry suggestive of hypopituitarism and has hyponatraemia is to establish whether he has hypoadrenalism. Thus the short synacthen test is most appropriate and in view of the seriousness of undetected hypoadrenalism should be the first test.
-
Question 21 of 100
21. Question
1 pointsA 48 year old man with newly diagnosed Type 2 diabetes presents for evaluation. He wants to know by what criteria the recommended treatments for his diabetes are decided. According to the ADA/EASD consensus, which principles is the one upon which therapeutic choices should be based?
Correct
Incorrect
Explanation:
The consensus authors state that ability to achieve and maintain glycaemic control is the most important factor when selecting between diabetes therapies, with:
-safety
-tolerability
-Side-effects
-ease of use and
-expense
being other factors of importance. -
Question 22 of 100
22. Question
1 pointsA 53 year old woman has had type 2 diabetes for past 3 years. She is currently treated with metformin 850 mg BD. On exam her BP is 152/85 mmHg and BMI is 29. HbA1c is 8.1% (<5.5). You are planning to add a Sulphonylurea to her regime. Which features, according to the consensus, is a key advantage or disadvantage of SU therapy?
Correct
Incorrect
Explanation:
The question of increased cardiovascular risk for sulfonylureas (SUs) was raised after the university group diabetes programme (UGDP) stud conducted during the 1970s. The consensus authors state however that the question ha: since been answered by the UK prospective diabetes study (UKPDS) and ADVANCE studies, where no cardiovascular risk signal was seen for SUs. Whilst SUs are well known to have a rapid onset of glucose lowering effect, their side effects of hypoglycaemia and weight gain are well own.
-
Question 23 of 100
23. Question
1 pointsA 24 year old woman has weight gain, oligomenorrhoea and primary infertility. She takes Lithium for bipolar disorder. On exam her BMI is 32kg/m2. Labs reveal free T4 of 6.4 pmol/L (10-22), TSH of 42 mU/L (0.4-5) and prolactin of 980 mU/L (50-450). The most appropriate treatment for this patient is
Correct
Incorrect
Explanation:
This patient has primary hypothyroidism which would explain the increasing weight and the associated hyperprolactinaemia. The latter occurs as a consequnece of reduced dopaminergic tone. The most appropriate treatment for her would be thyroxine which would through euthyroidism be expected to normalise prolactin concentration. In turn this may improve weight, menstrual function and fertility.
-
Question 24 of 100
24. Question
1 pointsAn 80-year-old non-diabetic suffers a seizure. Blood glucose is recorded as 1.0 mmol/L. He is given 50ml of 50% dextrose and he recovers in one hour. A serum cortisol concentration later returns as 800 nmol/L (120-600). Which of the following would be the most relevant investigation for this man?
Correct
Incorrect
Explanation:
The historical and biochemical evidence here suggests a diagnosis of spontaneous hypoglycaemia and the most likely cause would be an insulinoma. However, one would wish to exclude possible drug administration and although not mentioned here, a sulphonylurea screen should be undertaken. He has presented with symptomatic hypoglycaemia, is not diabetic therefore should not have received insulin or a sulphonylurea. There is nothing to suggest alcohol or drug misuse. Similarly, there is nothing to suggest sepsis. However, to prove a diagnosis of spontaneous hypoglycaemia a prolonged fast is required and he should develop hypoglycaemia, measurement of insulin and C-peptide will be needed to confirm the diagnosis. The appropriate cortisol response during his hypoglycaemia episode (cortisol 800) excludes hypoadrenalism.
-
Question 25 of 100
25. Question
1 pointsA 63-year-old female with a six year history of type 2 diabetes attends for annual review. Her HbA1c is 10% (3.8-6.4). Into what average plasma glucose concentration does her HbA1c translate?
Correct
Incorrect
Explanation:
The HbA1c is an important reflection of control over a 3 month period (life expectancy of the erythrocyte). There is a good relationship between the rise in glucose and its ability to glycosylate the Hb molecule (there´s a difference between average plasma glucose and blood glucose. Thus a HbA1c of 7% would translate into an average PLASMA (higher than value of BLOOD glucose) glucose of 9.5 mmol/L and a HbA1c of 10% into 15.5 mmol/L This is the reason why so much emphasis is placed on controlling HbA1c rather than the specific glucose measurements as these vary so much throughout the day.
-
Question 26 of 100
26. Question
1 pointsA 36-year-old male presents with weakness and tiredness. He is noted to be hypertensive. Electrolytes show a Hypokalaemia and a hypomagnesaemia. What investigation would you select for this patient?
Correct
Incorrect
Explanation:
The hypokalaemic hypertension with hypomagnesaemia suggests primary hyperaldosteronism. The most reliable assessment for this would be renin to aldosterone ratio.
-
Question 27 of 100
27. Question
1 pointsA 54 year old lady during the last year has stopped having periods and has had moderately severe sleep disturbance. She wakes up at night with sweats. She has a previous history of depression and her family history is significant for osteoporosis, heart disease, and Alzheimer´s disease in older members of her family. There is no family history of breast cancer. She wants to know about HRT. Which statement is correct about HRT?
Correct
Incorrect
Explanation:
Hormone replacement therapy (HRT) improves the urogenital symptoms of menopause, such as vaginal dryness and dyspareunia. However, recent research regarding HRT has not shown a benefit for reducing coronary events, slowing the progression of Alzheimer´s disease, improving depression or improving urinary incontinence.
-
Question 28 of 100
28. Question
1 pointsA 36 year old woman is brought to the ER in severe distress. She has had the “flu” for three days. Past history reveals a six month history of fatigue and malaise. Physical exam reveals BP 120/65 mmHg supine, 90/58 mm Hg standing, and darkened areas of skin on the knuckles, creases of the palm, elbows and an abdominal scar. Laboratory values are Na 122 mmol/L, K 5.8mmol/L, Cl 95 mmol/L, CO2 18mmol/L. The laboratory test that is most likely to assist in confirming your diagnosis is
Correct
Incorrect
Explanation:
Addison´s disease (adrenal insufficiency) is an insidious, usually progressive hypofunctioning of the adrenal cortex. It produces various symptoms, including hypotension and hyperpigmentation, and can lead to adrenal crisis with cardiovascular collapse. Diagnosis is clinical and is made by finding elevated plasma ACTH with low plasma cortisol. Treatment depends on the cause but generally includes hydrocortisone and sometimes other hormones.
-
Question 29 of 100
29. Question
1 pointsA 42 year old female has episodic headaches that last 30minutes and are associated with sweating, palpitations, and feelings of apprehension. She has lost 6.8 kg over the past three months. Physical examination reveals a thin woman with a pulse of 112 beats/minute and a blood pressure of 150/100 mmHg lying and 130/80mmHg standing. The most helpful diagnostic test is which of the following?
Correct
Incorrect
Explanation:
A pheochromocytoma is a catecholamine secreting tumor of chromaffin cells typically located in the adrenals. It causes persistent or paroxysmal hypertension. Diagnosis is by measuring catecholamine products in blood or urine. Imaging tests, especially CT or MRI, help localize tumors. Treatment involves removal of the tumor when possible. Drug therapy for BP control includes alpha blockade possibly combined with beta blockade.
-
Question 30 of 100
30. Question
1 pointsAn elderly woman notes the gradual onset of fatigue and feeling tired. She has gained a significant amount of weight over the last year. She frequently complains that the house is too cold and is turning up the thermostat. Constipation has become a problem. True statement regarding her condition is
Correct
Incorrect
Explanation:
Hypothyroidism is thyroid hormone deficiency. It is diagnosed by clinical features such as a typical facies, hoarse slow speech, and dry skin, and by low levels of thyroid hormones. Symptoms may include cold intolerance, constipation, forgetfulness, fatigue, personality changes and weight gain. Serum TSH is the most sensitive test. In primary hypothyroidism, there is no feedback inhibition of the intact pituitary, and serum TSH is always elevated, whereas serum free T4 is low. In secondary hypothyroidism, free T4 and serum TSH are low.
-
Question 31 of 100
31. Question
1 pointsA 36 year old female presents with palpitations, restlessness, sweating, weight loss, and a tremor for the past 6 months. Examination shows a fine tremor, lid lag and stare, and pretibial myxedema. The thyroid gland is diffusely enlarged, asymmetric, and lobular. A bruit is present over the gland. Investigations show an undetectable level of thyroid stimulating hormone, an increased level of thyroid hormones, and an increased radioactive iodine uptake. The diagnosis of Graves disease is made and the treatment options are discussed. The patient selects radioactive iodine therapy. This patient is at greatest risk for which of the following?
Correct
Incorrect
Explanation:
Hypothyroidism is this main complication of radioactive iodine therapy, affecting up to 70% of patients in 10 years. Radioactive iodine therapy is a safe and effective treatment for Graves´s disease because it can provide the same ablative effects of surgery without the surgical complications. There is no evidence that this treatment increases the risk for carcinoma.
-
Question 32 of 100
32. Question
1 pointsA 64 year old man presents with intermittent neurological abnormalities associated with hypoglycemia. The patient experiences an attack in the office, and the physician draw blood for serum insulin, C peptide, and glucose. The attack is relieved by giving the patient glucose. A week later, laboratory results reveal low serum glucose, high serum insulin and low serum C peptide. Which one of the following would result in these clinical and laboratory findings?
Correct
Incorrect
Explanation:
C-peptide is measured to differentiate insulin produced by the body from insulin injected into the body. When insulin is synthesized by the beta cells of the pancreas, it is produced as a large molecule (a propeptide). This molecule is then split into two pieces: insulin and C-peptide. The function of C-peptide is not known. The C-peptide level may be measured in a patient with type 1 diabetes to see if any insulin is still being produced by the body. It may also be measured in the evaluation of hypoglycemia to see if the person´s body is producing too much insulin. Normal values in a patient requiring insulin injections indicate that the person´s body is still producing some insulin. Normal values in a patient who has low blood sugar indicate that the patient is making too much insulin.
-
Question 33 of 100
33. Question
1 pointsThe treatment of choice for hypothyroidism is
Correct
Incorrect
Explanation:
Synthetic thyroid replacement via thyroxine is the treatment of choice for hypothyroidism. Iodide inhibits secretion of thyroid hormone, while methimazole inhibits synthesis. Ablation of the thyroid by radioactive hormone or surgery is contraindicated.
-
Question 34 of 100
34. Question
1 pointsThe bronze skin discoloration associated with hemochromatosis is due to deposition of
Correct
Incorrect
Explanation:
Melanin is responsible for the hyperpigmentation often seen in hemochromatosis. None of the other choices is correct.
-
Question 35 of 100
35. Question
1 pointsWhich of the following is NOT usually a major component in the treatment of diabetic ketoacidosis?
Correct
Incorrect
Explanation:
Oxygen is not usually necessary in uncomplicated diabetic ketoacidosis. All of the other answers are important parts of the treatment.
-
Question 36 of 100
36. Question
1 pointsA 56 year old female presents with difficulty swallowing. A thorough workup by a gastroenterologist fails to reveal any primary esophageal disease. The woman is referred to an endocrinologist. A biopsy of the thyroid reveals the infiltration with fibrous tissue. The most probable diagnosis is which of the following?
Correct
Incorrect
Explanation:
Riedel thyroiditis, also called ligneous (rocklike) stroma, is a rare form of chronic: thyroiditis characterized microscopically by a marked fibrous reaction that destroys most or all of the thyroid gland and may involve adjacent structures. The etiology is unknown. Clinically, this disease tends to affect middle-aged and older, mostly female patients and causes the thyroid to have a firm “woody” texture. It may be clinically mistaken for a neck malignancy and can cause symptoms of stridor, dyspnea, dysphasia, laryngeal nerve paralysis, or hypothyroidism.
Eyeball protrusion (choice A) suggests the hyperthyroidism of Graves´s disease. A, soft thyroid gland (choice B) suggests multinodular goiter.
A painful and tender thyroid (Choice C) suggests subacute granulomatous (de Quervain) thyroiditis.
A single large thyroid nodule (choice D) could be due to either a thyroid adenoma or thyroid cancer. -
Question 37 of 100
37. Question
1 pointsA 44 year old woman undergoes surgery because of a large thymic mass. The patient presented with chronic complaints of fluctuating muscle weakness predominanty affecting the extraocular muscles. The gross photograph below shows the surgical specimen. Histology reveals a mixture of spindle shaped epithelial cells arranged in whorls, and mature lymphocytes. The neuromuscular condition that was most likely associated with his thymic tumor is
Correct
Incorrect
Explanation:
The gross appearance of this large thymic mass, with its solid lobulated pattern, is compatible with a thymoma. Thymomas are composed of a variable mixture of epithelial cells and lymphocytes that more or less closely mimic the normal histologic components of normal thymus. Thymic hyperplasia and thymomas are strongly associated with myasthenia gravis, a neuromuscular condition characterized by weakness and fatigability affecting extraocular muscles and other muscles. This condition is mediated by a type II hypersensitivity reaction produced by autoantibodies that react with muscular nicotinic receptors and interfere with cholinergic neurotransmission. Duchenne muscular dystrophy is the most common of dystrophic myopathies. It is inherited as an X-linked condition, due to mutations of large gene-coding for a normal membrane protein (dystrophin). Guillain-Barré syndrome is characterized by ascending weakness beginning in the distal legs and spreading upward to involve upper limbs and respiratory muscles. It is due to an acute inflammatory demyelinating process affecting the spinal roots. It is often preceded by a viral infection and is thought to be mediated by an abnormal immune response. Myotonic dystrophy is an inherited disorder due to abnormal expansion of a trinucleotide repeat in a gene coding for myotonin protein kinase. The disease manifests with sustained contraction and stiffness of skeletal muscles. Baldness, cataracts, and cardiomyopathy are associated abnormalities. Toxic neuropathy may result from a number of toxic insults, including lead, organic compounds, and arsenic. It usually manifests with axonal degeneration and secondary muscle denervation.
-
Question 38 of 100
38. Question
1 pointsA 34 year old man presents with decreased libido. According to him he did not start shaving until he was 18 and now needs to shave only every other day. He has virtually no secondary sexual hair. His testes are small and atrophic and skin is soft and pale. Which of the following is the most likely diagnosis?
Correct
Incorrect
Explanation:
Non secreting chromophobe tumors of the pituitary are often associated with soft, smooth skin, absence of secondary sexual hair, and small and atrophic testes. Klinefelter´s syndrome (XXY) produces small testes, infertility and, often, gynecomastia. Secondary sexual hair is reduced. The skin may become quite wrinkled. A secretory pituitary tumor would not produce a clinical syndrome such as this, nor would pseudotumor cerebri (benign increased intracranial pressure) or Germinoma.
-
Question 39 of 100
39. Question
1 pointsA 30 year old woman has sudden convulsions and paresthesias of the lips and extremities. She also has extensive spasm of her (arms, causing cramps. There is contraction of the facial muscle in response to tapping the facial nerve against the bone anterior to the ear. ECG shows a prolonged QT interval. Which of the following is NOT associated with this condition?
Correct
Incorrect
Explanation:
This patient has symptoms of hypocalcemia. This patient has Chvostek´s sign with spasm of the facial muscle. Prolongation of the QT interval can cause torsades de pointes, a ventricular arrythmia. Sarcoidosis causes hypercalcemia via an increase in hydroxylation of vitamin D by the granulomas and, thus, an increase in calcium absorption. Chronic renal insufficiency (A) causes hypocalcemia because of inadequate 1-hydroxylation of vitamin D by the kidneys and thus a decrease in absorption of calcium via the GI tract. Hypoparathyroidism (B) is secondary to decrease in parathyroid hormone and, thus, decrease in bone resorption and hypocalcemia. Malabsorption (C) causes hypocalcemia via decreased absorption via the GI tract. Examples are pancreatic insufficiency and gluten-sensitive enteropathy. Sepsis (E) causes 20% of hypocalcemic symptoms in Gram-negative sepsis. These patients have a high mortality rate. This is secondary to defects in the parathyroid-vitamin D axis.
-
Question 40 of 100
40. Question
1 pointsMixed metabolic acidosis and alkalosis is most likely seen in the setting of which of the following?
Correct
Incorrect
Explanation:
An individual with diabetic acidosis and vomiting is most likely to have a mixed metabolic acidosis and alkalosis. The vomiting expels hydrochloric acid from the stomach, producing a metabolic alkalosis. Severe pulmonary disease produces metabolic alkalosis and respiratory acidosis. Lactic acidosis produces a pure metabolic acidosis. Lactic acidosis combined with diabetic acidosis similarly produces metabolic acidosis. Mechanical ventilation can result in respiratory alkalosis.
-
Question 41 of 100
41. Question
1 pointsA 47 year old man has symptoms of refractory hypertension, hypokalemia, and arteriolar narrowing on funduscopic exam. His blood pressure is not controlled by antihypertensives. No abdominal bruit or hyperpigmentation was observed. Investigations revealed a low plasma renin level. The most likely diagnosis is
Correct
Incorrect
Explanation:
Symptoms of primary aldosteronism are caused by excessive aldosterone secretion from the adrenal cortex either from adrenal hyperplasia or an adrenal adenoma. Continued elevated aldosterone levels cause hypokalemia and diastolic hypertension. Diuretics exacerbate the symptoms of hypokalemia. Renin levels are usually suppressed. Secondary hyperaldosteronism results from stimulation of the adrenal glomerulosa by extra adrenal factors. The renin level is usually elevated. Renal artery stenosis is another possible cause for secondary hypertension. This patient has no abdominal bruits to suggest renal artery stenosis. The potassium level would be normal, not low. Pheochromocytoma patients would present with palpitations, fluctuating blood pressure levels, and diaphoresis. The potassium level is usually normal. The CT scan may detect the tumor in the adrenal medulla. Cushing´s´ disease would result in hypertension secondary to excess corticosteroids in the body. This disorder is more common in females. The patient would also be obese with osteoporosis, striae, and moon faces.
-
Question 42 of 100
42. Question
1 pointsWhich of the following does NOT result in thyrotoxicosis?
Correct
Incorrect
Explanation:
131I therapy, used to treat hyperthyroidism, may result in hypothyroidism. Thyrotoxicosis describes the clinical syndrome resulting from excess thyroid hormone. Increased thyroid hormone production may result from increased occupancy of the TSH receptor by thyroid stimulating immunoglobulin in Graves´ or Hashimoto´s disease. These receptors may also be occupied by human chorionic gonadotropin produced by hydatiform moles or choriocarcinoma. The Jodbasedow effect, occurring primarily in persons with toxic nodular goiter, designates iodine induced hyperthyroidism and may result in thyrotoxicosis. Ectopic production of thyroid hormone may occur in ovarian teratomas.
-
Question 43 of 100
43. Question
1 pointsA 50 year old woman has increased serum calcium during routine checkup. She has no specific complaints but she feels weak recently and has experienced an increased incidence of constipation. Serum phosphate is also below normal and urinary cAMP levels are increased. What is the likely diagnosis?
Correct
Incorrect
Explanation:
Primary hyperparathyroidism is often asymptomatic and is only incidentally discovered during routine blood work; however, there may be vague complaints of fatigue or weakness and constipation. These neuromuscular manifestations are due to the hypercalcemia, which can “hyperstabilize” excitable tissue membranes and reduce normal responsiveness. The incidence of primary hyperparathyroidism increases greatly after age 50, and it is more common in women than men. The hypercalcemia is due to the excess plasma concentration of parathyroid hormone (PTH). In primary hyperparathyroidism, 80% of the cases are due to a single adenoma in a parathyroid gland that secretes excessive PTH. In the other 20% of cases the hypersecretion of PTH is due to hyperplasia in multiple parathyroid glands. The increased PTH also causes renal excretion of phosphate producing hypophosphatemia. PTH acts by increasing cAMP formation in target tissues. The cAMP formed in renal tubules can diffuse into the lumen and be measured in the urine. Tumors that secrete calcitonin include medullary carcinema of the thyroid and occasionally small and large cell carcinomas of the lung. Despite the high blood concentration of calcitonin, serum calcium and phosphate are rarely abnormal. Primary hypoparathyroidism, which is due to decreased secretion of PTH is associated with hypocalcemia and hyperphosphatemia. Furthermore, urinary cAMP concentration would be decreased. The low calcium in extracellular fluid “destabilizes” excitable tissue membranes and can lead to spontaneous action potentials that produce tetany. Pseudohypoparathyroidism is a rare genetic disorder that results in hypocalcemia and hyperphosphatemia due to end organ unresponsiveness to PTH. With vitamin D deficiency serum calcium is decreased due to diminished absorption from the diet. PTH secretion is increased to compensate, resulting in bone demineralization (osteomalacia).
-
Question 44 of 100
44. Question
1 pointsA diagnosis of diabetes mellitus is being considered in 32-year-old woman who is 16 weeks pregnant. Her BMI was 22 kg/m2 (18 -25). A 75g oral glucose tolerance test revealed:
Time Plasma glucose concentration 0 hr 6.0 mmol/l (3.0-6.0) 2hr 12.5 mmol/l (<11.1) What is the next most appropriate step in her management?
Correct
Incorrect
Explanation:
The result confirms a diagnosis of gestational diabetes mellitus with the 2hr OGTT result above 11.1mmol/1. To minimize the fetal consequences of GDM (macrosomia, fetal malformations, still birth, IUGR etc), the patient´s glycaemia should be strictly controlled with insulin. A low calorie diet is inappropriate and neither metformin nor glipizide are licenced for use in pregnancy. There is no point in repeating the OGTT in 4 weeks as control is required NOW.
-
Question 45 of 100
45. Question
1 pointsA 44 year old previously healthy female presents complaining of severe headache and vomiting for 12 hours. She takes no medication. Her temperature is 37.5°C, pulse rate is 110/min and BP is 95/60 mmHg. There is neck stiffness and a right third nerve palsy with papillary involvement. She has hyponatremia and hyperkalemia and low random cortisol levels. The most likely diagnosis is
Correct
Incorrect
Explanation:
This patient has had an acute onset of headache secondary to haemorrhage into the pituitary gland. She has evidence of hypopituitarism given her low random cortisol, consistent biochemical results of hyponatraemia and hyperkalaemia. Her third cranial nerve palsy is due to enlargement of the pituitary producing compression of the third cranial nerve. She requires resuscitation and steroid replacement, urgent imaging of the pituitary gland and consideration of neurosurgical decompression.
-
Question 46 of 100
46. Question
1 pointsA 45 year old obese male with a two year history of type 2 diabetes has recently started metformin at a dose of 500 mg twice daily. However, he reports numerous gastrointestinal side effects including bloating and flatulence. He is keen to commence an alternative agent. The most appropriate choice is
Correct
Incorrect
Explanation:
An obese type 2 diabetic patient is almost certain to be insulin resistant. Current guidelines suggest that first line therapy in obese, insulin resistant patients should be an insulin sensitising agent (Bailey C et al. Br J Diabetes vas Dis 2006; 6: 147-148). Metformin is the drug of choice in this patient therefore.
However, in patients intolerant of metformin, the thiazolidinediones rosiglitazone and pioglitazone are licensed for monotherapy for insulin resistant patients.
Previously, sulphonylureas, such as gliclazide, would have been advocated in these circumstances, but current opinion favours rosiglitazone and pioglitazone in such a patient. The recent joint ADA / EASD guidelines for treatment in Type 2 diabetics suggests the glitazones are a useful choice where hypoglycaemia is to be avoided. Glitazones increase the risk of congestive heart failure. There is currently controversy over rosiglitazone since some data suggest it is associated with an increased risk of ischaemic heart disease. -
Question 47 of 100
47. Question
1 pointsA 41 year old female, with no prior history of thyroid disease presents with a 5 day history of an acutely painful, left sided goitre. Clinically she appeared euthyroid, and was apyrexial. Her labs are as follows: Haemoglobin 13.0 g/dL, WBC count 7.0 x109/L and platelet count 200 x109. The most likely diagnosis is
Correct
Incorrect
Explanation:
The left side of this patient´s goitre becomes acutely swollen with no other signs and FBC is normal which suggests acute haemorrhage into a cyst. Thyroid cancer is usually painless. De Quervain´s thyroiditis is a diffusely tender goitre typically with systemic features such as Weight loss, pyrexia and a raised ESR.
-
Question 48 of 100
48. Question
1 pointsA 46 year old teacher presents with fatigue. She is noted to be hypercalcaemic with an albumin of 39 g/L (37-49), globulin of 28g/L and Ca++ of 2.80mmol/l (2.2-2.6). True statement regarding her is which one of the following?
Correct
Incorrect
Explanation:
Twenty four hour urinary calcium may be useful if used in comparison the serum calcium in order to distinguish familial hypocalciuric hypercalcaemia from primary hyperparathyroidism. Parathyroid hormone (PTH) may be less than twice the upper limit of normal in primary hyperparathyroidism.
Several indications for surgery exist including:- Calcium more than1 mg/dl above upper normal limit
- Greater than 30% decline in renal function
- Renal stones
- Age less than 50
- Unwillingness of patient to follow advice of medical surveillance. Myeloma is unlikely given the normal immunoglobulins.
-
Question 49 of 100
49. Question
1 pointsA 55 year old man is suspected to have acromegaly. The best investigation to confirm the diagnosis is which one of the following?
Correct
Incorrect
Explanation:
The diagnosis of Acromegaly is confirmed by inadequate suppression of GH concentrations below 2mU/l in an oral glucose tolerance test. Although IGF-1 concentrations are elevated these are not diagnostic.
-
Question 50 of 100
50. Question
1 pointsA 46 year old diabetic Asian female has a long history of weakness and fatigue. Recently she has difficulty ascending stairs at home. Initial X-rays reveal healing clavicular fractures, and a superior public rami fracture. Labs show hypocalcemia and hypophosphatemia with elevated ALP level. HbA1c is 11.0%. What is the likely diagnosis?
Correct
Incorrect
Explanation:
This patient has vitamin D deficiency. It classically presents in the female Asian population whose clothing offers little exposure to sunlight. The phosphate and calcium are usually low normal, and the alkaline phosphatase is high.
Bone deformity or rickets may develop in children. Fractures can occur due to bone demineralisation with osteoporosis on DEXA. Proximal myopathy is often a presenting feature of osteomalacia as is probably the case with this patient. -
Question 51 of 100
51. Question
1 pointsWhich finding would be expected in active acromegaly with associated diabetes mellitus?
Correct
Incorrect
Explanation:
Insulin resistance stems from the excessive growth hormone (GH) concentrations (anti-insulin effects) that of course fail to suppress with hyperglycaemia.
Acromegaly is often effectively treated with somatostatin analogues which may improve glycaemic control. Many of the effects of GH are mediated through IGF- 1, concentrations of which are high in acromegaly. Diabetes mellitus is due to the insulin resistance and is not due to auto-immune insulinitis. -
Question 52 of 100
52. Question
1 pointsWhich one of the following is a side effect of recombinant human growth hormone (rhGH)?
Correct
Incorrect
Explanation:
Unlike the old pituitary derived growth hormone (GH), rhGH is not associated with CJD as it is manufactured by recombinant techniques. rhGH therapy has been associated with BIH probably due to the fluid retention associated with GH therapy.
-
Question 53 of 100
53. Question
1 pointsA 70-year-old female presents with a six month history of frontal headaches and weight loss. On examination a bitemporal hemianopia was noted. Which of the following suggest the diagnosis of a pituitary tumour?
Correct
Incorrect
Explanation:
The raised prolactin would most likely reflect stalk compression in this patient. Otherwise, the normal cortisol would be unhelpful as is the normal thyroid-stimulating hormone (TSH). The elevated luteinising hormone (LH) is a reflection of this patient being menopausal. Growth hormone (GH) concentrations are frequently undetectable as they are released episodically usually during the night.
-
Question 54 of 100
54. Question
1 pointsA 54-year-old male presents with a 3 months history of polyuria with polydipsia. Which of the following measurements would confirm a diagnosis of diabetes mellitus?
Correct
Incorrect
Explanation:
Diabetes Mellitus is diagnosed on the basis of symptoms plus a random glucose above 11.1mmol/1 or fasting plasma glucose above 7mmol/l or the two hour oral glucose tolerance test. Impaired glucose tolerance would be indicated by a post OGTT plasma glucose between 7.7 and 11.1 or a fasting plasma glucose between 6.1 and 7.
-
Question 55 of 100
55. Question
1 pointsA 50 year old woman with recently diagnosed hypertension presents to her family physical complaining of hirsutism acne. Laboratory examination reveals cortisol 1500 nmol/L (N: 200-660 nmol/L) aldosterone 200 pmol/L (N: 140- 415 pmol/L) ACTH 4 pmol/L (N: 4 -22pmol/L). The most likely diagnosis is
Correct
Incorrect
Explanation:
In Cushing´s syndrome, the level of corticosteroids is excessive; usually from overproduction by the adrenal glands. Corticosteroids alter the amount and distribution of body fat. Excessive fat develops throughout the torso and may be particularly noticeable at the top of the back. A person with Cushing´s syndrome usually has a large, round face (moon face). The arms and legs are usually slender in proportion to the thickened trunk. Muscles lose their bulk, leading td weakness.
High corticosteroid levels over time raise the blood pressure, weaken bones (osteoporosis), and diminish resistance to infections. The risk of developing kidney stones and diabetes is increased and mental disturbances, including depression and hallucinations, may occur. Women usually have an irregular menstrual cycle. Children with Cushing´s syndrome grow slowly and remain short. In some people, the adrenal glands also produce large amounts of androgens (testosterone and similar hormones), leading to increased facial and body hair in women and balding. When doctors suspect Cushing´s syndrome, they measure the level of cortisol, the main corticosteroid hormone, in the blood. Normally, cortisol levels are high in the morning and lower late in the day. In people who have Cushing´s syndrome, cortisol levels are very high throughout the day. -
Question 56 of 100
56. Question
1 pointsOut of the following, which is associated with thyroid disease?
Correct
Incorrect
Explanation:
Vitiligo is a loss of skin melanocytes that causes areas of skin depigmentation of varying sizes. Cause is unknown, but the condition may be autoimmune, as up to 1/3 of patients have evidence of other autoimmune disease. Vitiligo affects 0.5 to 2% of the population. Etiology is unknown, but melanocytes are lacking in affected areas; some patients have antibodies to melanin. Up to 30% have other autoimmune antibodies (to thyroglobulin, adrenal cells, and parietal cells) or clinical autoimmune endocrinopathies (Addison´s disease, diabetes mellitus, pernicious anemia, and thyroid dysfunction), leading to speculation that vitiligo is an autoimmune disease. However, the relationship is unclear and may be coincidental. The strongest association is with hyperthyroidism (Graves´ disease) and hypothyroidism (Hashimoto´s thyroiditis).
-
Question 57 of 100
57. Question
1 pointsThe diabetes drug that is contraindicated in liver and kidney failure is
Correct
Incorrect
Explanation:
Insulin works by directly pushing glucose from the bloodstream into the cells. Metformin works by inhibiting hepatic gluconeogenesis. Glyburide is an oral sulfonyl urea hypoglycemic drug that increases pancreatic insulin secretion. Acarbose inhibits enzymes needed to digest carbohydrates. Because the carbohydrates are not broken down into glucose molecules, less glucose is absorbed into the bloodstream. Metformin is contraindicated in people with any condition that could increase the risk of lactic acidosis, including liver and kidney disorders (creatinine levels over 150µmol/I).
-
Question 58 of 100
58. Question
1 pointsA 33 year old man is found to have a blood pressure of 180/105 mmHg. He admits to stress at work and states that his father was diagnosed as hypertensive at the age of 60 but, thus far, has not required treatment. He denies taking any medications. He smokes 25 cigarettes per day but takes no alcohol. He is not obese, has no signs of cardiovascular disease, and no hypertensive retinopathy. The results of investigation are given below:
Urine analysis negative
Na 146 mmol/L
K 2.8 mmol/L
C1 102 mmol/L
HCO3 33 mmol/L
Urea 4.2 mmol/L
Creatinine 92 mmol/L
Glucose 6.2 mmol/L
Which of the following investigations is indicated in this patient?Correct
Incorrect
Explanation:
In primary hyperaldosteronism (Conn´s disease), overproduction of aldosterone leads to fluid retention and increased blood pressure, weakness, and, rarely, periods of paralysis.
Hyperaldosteronism can be caused by a tumor in the adrenal gland or may be a response to some diseases. High aldosterone levels can cause high blood pressure and low; potassium levels; low potassium levels may cause weakness, tingling, muscle spasms, and periods of temporary paralysis. Doctors measure the levels of sodium, potassium, and aldosterone in the blood. Sometimes, a tumor is removed, or people take drugs that block the action of aldosterone.
Doctors who suspects hyperaldosteronism first tests the levels of sodium and potassium in the blood. Doctors may also measure aldosterone levels. If they are high, spironolactone or eplerenone, drugs that block the action of aldosterone, may be given to see if the levels of sodium and potassium return to normal. In Conn´s syndrome, the levels of renin are also very low. When too much aldosterone is being produced, doctors examine the adrenal glands for a noncancerous tumor (adenoma). Computed tomography (CT) or magnetic resonance imaging (MRI) can be helpful, but sometimes blood samples from each of the adrenals must be tested to determine the source of the hormone. -
Question 59 of 100
59. Question
1 pointsHyperosmolar nonketotic diabetic coma is most common in
Correct
Incorrect
Explanation:
This condition is most commonly seen in the elderly. It is usually found in diabetics between the ages of 50 and 70. It is not more common in patients with previous episodes of ketoacidosis, adolescents, pregnant women, or children.
-
Question 60 of 100
60. Question
1 pointsA 55 year old diabetic woman has had a lingering sore throat and has lost weight because of pain on swallowing. Her condition would respond well to which of the following?
Correct
Incorrect
Explanation:
A picture such as this in a diabetic individual is very likely to be thrush, i.e., candida, which usually responds to ketoconazole. None of the other choices is appropriate.
-
Question 61 of 100
61. Question
1 pointsA 37 year old woman presents with symptoms of lethargy constipation, cold intolerance, weight gain, and menorrhagia. Physical examination shows dry skin, periorbital puffiness, macroglossia, and prolonged relaxation phase of the deep tendon reflexes. The most sensitive test to make the diagnosis is
Correct
Incorrect
Explanation:
Thyroid stimulating hormone is secreted by the pituitary gland, and it is the most sensitive test to detect and diagnose hypothyroidism. The third generation assay for TSH is available. Hashimoto´s thyroiditis is the most common cause for hypothyroidism. The T3 resin uptake (A) measures the unoccupied plasma binding sites for T4 and T3 and thereby provides an indirect assessment of free hormone concentration. A low T3 RU indicates a high binding capacity and is seen in hypothyroidism, pregnancy and estrogen administration. This test is now outdated and clinicians use the TSH instead. Anti-microsomal antibodies (B) and thyroglobulin antibodies are present in autoimmune thyroid states such as Hashimoto´s thyroiditis and Grave´s disease. Serum thyroglobulin (D) is increased in patients with an enlarged thyroid or following acute trauma to the thyroid. Thyroglobulin determinations are most useful in the follow-up of patients with metastatic thyroid carcinoma following thyroidectomy. Reverse T3 (E) derives from the peripheral metabolism of T4 to T3. Its measurement is not generally useful clinically.
-
Question 62 of 100
62. Question
1 pointsHypocalcemia is NOT associated with which one of the following?
Correct
Incorrect
Explanation:
Hypocalcemia is not associated with ingestion of Aminophylline. It can be a manifestation of magnesium deficiency (A) pancreatitis (B), hypoparathyroidism (C), and acute rhabdomyolysis (D).
-
Question 63 of 100
63. Question
1 pointsA 29 year old woman comes complains of fatigue, weakness and palpitations. She is divorced and lives with her 4 year old daughter. Complete evaluation show that this patient has hyperthyroidism and mild ophthalmopathy caused by Graves´s disease. Before initiating therapy, the patient wants to know what she can expect in the future. What would be the correct advice for her prognosis?
Correct
Incorrect
Explanation:
Hyperthyroid patients are at increased risk for cardiac arrhythmias, typically tachyarrhythmias. Graves´ ophthalmopathy follows a course that is independent of thyroid disease. Malignant degeneration is not a complication of Graves´s disease. Hyperthyroidism does not preclude the ability to become pregnant. There is no evidence to suggest that every Graves thyroid increases in size without surgery.
-
Question 64 of 100
64. Question
1 pointsWhich of the following does NOT decrease the progression of diabetic nephropathy in long standing type I diabetes mellitus?
Correct
Incorrect
Explanation:
Diabetic nephropathy can eventually progress to end stage renal disease. A low sodium diet is useful to control essential hypertension and its consequence, but not to prevent diabetic nephropathy from progressing. The DCCT trail has shown that tight control of diabetes and glucose control can slow down the progression of renal dysfunction in a type I diabetics. ACE inhibitors such as captopril can vasodilate the afferent arteroiles in the kidneys and thus prevent glomeruler hypertrophy and also decrease the perfusion pressure. Radiocontrast dye used in certain tests can cause interstitial damage and thus damage the kidneys further. Other nephrotoxic drugs, such as aminoglycosides, should be used carefully in a diabetic with nephropathy.
-
Question 65 of 100
65. Question
1 pointsWhich of the following is NOT an approved use of estrogen replacement therapy?
Correct
Incorrect
Explanation:
Estrogen receptors are distributed widely in body tissues, including the brain. Limited evidence suggests that estrogen and progesterone modulate central nervous system neuronal activity and that estrogen deficiency contributes to the neurodegenerative changes of aging. Some studies suggest that estrogen replacement therapy may protect against Alzheimer´s disease in women and may improve cognitive performance of women with this illness. There is still not enough evidence to recommended estrogen-replacement therapy for this reason alone. Estrogen is the agent of choice for preventing postmenopausal bone loss and also for risk reduction for hip fracture up to 50%. Estrogen replacement therapy is useful in vaginal atrophy with associated dysuria, pruritis, incontinence, and dyspareunia. Estrogen indisputably is effective in controlling vasomotor complaints, such as hot flashes and night sweats. These symptoms are experienced by 75-85% of climacteric women and may persist for months to years. Estrogen replacement provides a cardioprotective effect, which may be a result of reduced LDL cholesterol, increased HDL cholesterol, reduced plasma fibrinogen levels, or increased fibrinolytic potential. Epidemiologic studies have estimated that estrogen reduces the risk of ischemic heart disease by as much as 50%. -
Question 66 of 100
66. Question
1 pointsA 52-year-old woman presented with nocturia and Pruritis vulvae. Urine dipstick analysis shows 2% glucose.Which one of the following would most reliably confirm a diagnosis of diabetes mellitus?
Correct
Incorrect
Explanation:
The post-prandial glucose gives an equivalent to the 75g glucose tolerance test, which would be one of the investigations of choice in this patient. None of the other results listed here formally confirm the diagnosis of Type 2 diabetes.
-
Question 67 of 100
67. Question
1 pointsWhich of the following techniques would be most useful in the differential diagnosis between ectopic Cushing´s syndrome and pituitary dependent Cushing´s disease?
Correct
Incorrect
Explanation:
Inferior petrosal sinus sampling with an elevated central adrenocorticotropic hormone (ACTH) concentration compared with the peripheral value is the most valuable test in the differential diagnosis of either cushing´s disease or ectopic Cushing´s syndrome. The other tests are far less useful in comparison.
-
Question 68 of 100
68. Question
1 pointsLow uptake of I-123 on the thyroid uptake scan would be an expected finding in:
Correct
Incorrect
Explanation:
DeQuervain´s thyroiditis is classically associated with low or absent I-123 (or I-131 radioactive isotopes of iodine) uptake. The others will have high or normal uptake. In particular type 1 amiodarone induced thyrotoxicosis may be distinguished from the thyroiditis of type 2 by the normal or high uptake scan.
-
Question 69 of 100
69. Question
1 pointsA 55-year-old with a known history of Graves´s disease presents with palpitations, anxiety and fine tremor of both hands. ECG shows rapid atrial fibrillation and ventricular rate of 160 to 180.min Blood pressure was 110/80 mmHg.
TSH 0.01 mU/l (0.4-5.0 mU/l) Free T4 4.69 ng/dl (0.8-1.8 ng/dl) What is the immediate management for this patient?
Correct
Incorrect
Explanation:
AF occurs in 10% to 25% of patients with hyperthyroidism, more commonly in men and the elderly than in women or patients less than 75 years old.
In a patient with hyperthyroidism and AF, initial therapy should focus on ventricular rate control with B-blockers, but conversion to sinus rhythm frequently occurs spontaneously with treatment of hyperthyroidism.
AF in thyrotoxicosis is characterised by rapid ventricular response, typically resistant to digoxin therapy.
Electric or pharmacologic cardioversion should be attempted only in euthyroid patients who are hemodynamically unstable.
If AF persists, consideration should be given to anticoagulation in patients who are at risk of embolic events. -
Question 70 of 100
70. Question
1 pointsA 74-year-old male with type 2 diabetes requires controlling his diabetes. He also has heart failure for which he is on a diuretic and an ACE inhibitor. Which of the following hypoglycemic agents is contraindicated in this patient?
Correct
Incorrect
Explanation:
Rosiglitazone can result in fluid retention of unknown etiology which may cause a mild dilutional anaemia (haemoglobin typically falls by 1 to 2 g/dL) and ankle edema.
It is contraindicated in hepatic dysfunction and congestive heart failure.
Sulphonylureas are contraindicated in renal failure. All should be stooped and insulin started if serum creatinine exceeds 250µmol/L.
The recent United Kingdom Prospective Diabetes Study found no evidence events that patients treated with sulphonylureas suffered cardiovascular events more often than those treated with insulin.
Contraindications of metformin include all the major organ failures:
– Renal
– Hepatic
– Cardiac and
– Respiratory
It should not be used when serum creatinine concentration exceeds 150µmol/L.
Side-effects of acarbose are due to carbohydrate malabsorption (flatus, abdominal bloating and gassy diarrhea). -
Question 71 of 100
71. Question
1 pointsA 69 year old woman is hospitalized for pneumonia. She appears acutely ill and slightly lethargic. Her examination is consistent with right lower lobe pneumonia but is otherwise normal. Her pulse rate is 90 beats/min and regular. Her weight is normal for height. Her TSH level is 9.0microU/mL (N 1.0-5.0). What is the most appropriate initial step for managing her thyroid abnormality?
Correct
Incorrect
Explanation:
The likelihood that this patient has significant thyroid disease is very low given the minimal elevation of TSH, normal clinical examination, and concomitant pneumonia. The elevated TSH level is likely due to her illness rather than to any underlying psychiatric condition. Even if she had a palpable thyroid, her risk of hypothyroidism would be on the order of 5%. A TSH level ≥ 20 U/mL in an acutely ill patient reflects true hypothyroidism only about 49% of the time. It is likely that this patient has sick euthyroid syndrome and that follow-up thyroid testing after discharge when she has recovered is appropriate and is very likely to be normal. Free T4, rT3, and TSH levels would be appropriate for subsequent evaluation if the patient´s laboratory values did not return to normal after resolution of the pneumonia.
-
Question 72 of 100
72. Question
1 pointsA 65 year old man presents with a history paroxysms of sweating, palpitations, headaches and anxiety. This happens off and on. His blood pressure on exam is 156/95 mmHg. On further history he tells you that one of his relatives had thyroid cancer. What tests would be appropriate to perform first at this time?
Correct
Incorrect
Explanation:
This patient needs to be screened pheocromocytoma. Pheochromocytoma is a tumor of the adrenal gland which causes very high levels of the catecholamines (epinephrine and norepinephrine) to be secreted into the bloodstream. This can lead to many sympathetic nervous system symptoms like elevated blood pressure, palpitations, anxiety, diaphoresis, headaches, and weight loss. These symptoms happen in spurts or paroxysms.
Diagnosis is made by measuring the level of the catecholamines and their breakdown products or metabolites which are called metanephrines in a 24 hour urine collection. Treatment involves medicines to control the blood pressure and surgery to remove the tumor. Recall the MEN syndromes.
MEN (Multiple endocrine neoplasia) has three types:
MEN I (Warmer syndrome): Tumors of the pancreas, pituitary and parathyroid.
MEN IIa (Sipple syndrome): Medullary thyroid carcinoma, pheochromcytoma and tumor of the parathyroid.
MEN IIb: Medullary thyroid carcinoma, pheochromocytoma and neuromas. This patient could have MEN II if you consider his family history. -
Question 73 of 100
73. Question
1 pointsThe best test in the long term follow up of a diabetic patient is which of the following?
Correct
Incorrect
Explanation:
HbA1C is a test that measures the amount of glycosylated hemoglobin in your blood. The test gives a good estimate of how well diabetes is being managed over time. In particular over the past 3 months, since that is the life span of the red blood cell that contains the hemoglobin molecule.
This test measures blood sugar control over an extended period in people with diabetes. In general, the higher your HbA1C value, the higher the risk that you will develop complications from diabetes (eye disease, kidney disease. nerve damage, heart disease, and stroke). This is especially true if your HbA1C remains elevated on more than one occasion. Most physicians will consider HbA1C less than 7 as an indicator of good diabetic control. Unlike a diary, the HbA1C value does not lie. -
Question 74 of 100
74. Question
1 pointsA 50 year old male who smokes two packs of cigarettes a day presents with a lung mass on X-ray and recent weight gain. Laboratory investigation shows hyponatremia with hyperosmolar urine. The most likely diagnosis in this patient is
Correct
Incorrect
Explanation:
SIADH (syndrome of inappropriate secretion of anti-diuretic hormone) is a common paraneoplastic syndrome that affects the endocrine system. This syndrome is most often associated with small cell lung cancer; however, other cancers such as brain tumors, leukemia, lymphoma, colon, prostate, and head and neck cancers can lead to SIADH. SIADH is caused by the inappropriate production and secretion of arginine vasopressin or antidiuretic hormone (ADH) by tumor cells. Patients with SIADH may not have symptoms, especially in the early stages. When symptoms do occur they are usually related to hyponatremia, which leads to central nervous system toxicity if left untreated. Lab values will show concentrated urine in the presence of plasma hypo-osmolality (hyponatremia).
Signs and symptoms associated with hyponatremia include fatigue, anorexia, headache and mild alteration in mental status in early stages. If SIADH remains untreated, symptoms can progress to confusion, delirium, seizures, coma, and death. Treatment approaches for SIADH are to treat the underlying tumor and restriction of fluids. More severe cases may require the administration of medications. -
Question 75 of 100
75. Question
1 pointsA 72 year old woman comes to you for the first time. She has taken levothyroxine (Synthroid), 0.3 mg/day, for the last 20 years. Although a recent screening TSH was fully suppressed at <0.1 micro U/mL, she claims that she has felt ´awful” when previous physician have attempted to lower her dosage. You explain that a serious potential complication of her current thyroid medication is which of the following?
Correct
Incorrect
Explanation:
Women older than 65 years of age who have low serum TSH levels, indicating physiologic hyperthyroidism, are at increased risk for new hip and vertebral fractures. Use of thyroid hormone itself does not increase the risk of fracture if TSH levels are normal.
-
Question 76 of 100
76. Question
1 pointsOut of the following, which is associated with thyroid disease?
Correct
Incorrect
Explanation:
Vitiligo is a loss of skin melanocytes that causes areas of skin depigmentation of varying sizes. Cause is unknown, but the condition may be autoimmune, as up to 1/3 of patients have evidence of other autoimmune disease. Vitiligo affects 0.5 to 2% of the population. Etiology is unknown, but melanocytes are lacking in affected areas; some patients have antibodies to melanin. Up to 30% have other autoimmune antibodies (to thyroglobulin, adrenal cells, and parietal cells) or clinical autoimmune endocrinopathies (Addison´s disease, diabetes mellitus, pernicious anemia, and thyroid dysfunction), leading to speculation that vitiligo is an autoimmune disease. However, the relationship is unclear and may be coincidental. The strongest association is with hyperthyroidism (Graves´ disease) and hypothyroidism (Hashimoto´s thyroiditis).
-
Question 77 of 100
77. Question
1 pointsIn the course of DKA, serum potassium levels
Correct
Incorrect
Explanation:
In the course of DKA, serum potassium levels can appear normal but total body potassium may actually be low. The reason is that in states of acidosis, protons exchange for K Ions In the cells. As the protons enter the cells, K Ions will leave the cells and enter the bloodstream. This will make it seem like the serum body potassium is normal.
-
Question 78 of 100
78. Question
1 pointsThe diagnostic test choice to confirm Cushing syndrome is
Correct
Incorrect
Explanation:
Cushing´s syndrome is a constellation of clinical abnormalities caused by chronic high blood levels of cortisol or related corticosteroids. Cushing´s disease is Cushing´s syndrome that results from excess pituitary production of ACTH, usually secondary to a pituitary adenoma. Typical symptoms Include moon facies and truncal obesity with thin arms and legs. Diagnosis is by history of receiving corticosteroids or by elevated serum cortisol. Diagnosis is confirmed with the dexamethasone test, in which 1, 1.5, or 2 mg of dexamethasone is administered po at 11 to 12 pm and plasma cortisol is measured at 8 to 9 am the next morning. In most normal patients, this drug suppresses morning plasma cortisol to ≤ 1.8 µg/mL (≤ 50 nmol/L), whereas patients with Cushing´s syndrome virtually always have a higher level. A more specific but equally sensitive test is to give dexamethasone 0.5 mg po q 6 h for 2 days (low dose). In general, a clear failure to suppress levels in response to low-dose dexamethasone establishes the diagnosis. -
Question 79 of 100
79. Question
1 pointsA 55-year-old schoolteacher attends with weight loss and sweats. She is clinically thyrotoxic with a diffuse goiter. Investigations show free T4=40 pmol/l, free T3=9.8 nmol/l and TSH=6.1 mU/l. A repeat TFT is similar. What is the most appropriate investigation for this patient?
Correct
Incorrect
Explanation:
This patient is thyrotoxic; however, as the non-suppressed thyroid- stimulating hormone (TSH) suggests that this is due to excessive TSH production by the pituitary gland, the possibility of a thyrotroph adenoma must be pursued.
In primary hyperthyroidism the TSH should always be suppressed by negative feedback, which is not the case here. TSH-omas are indeed very rare, but the giveaway would be the normal or elevated TSH with thyrotoxicosis. -
Question 80 of 100
80. Question
1 pointsA 46-year-old lady is found to have Thymoma. Which of the following conditions is associated with Thymoma?
Correct
Incorrect
Explanation:
“Originally, thymoma was cited as the primary cause of acquired pure red cell aplasia. However, subsequent studies revealed that thymomas caused only 2 to 37 cases of pure red cell aplasia. Conversely, only 7% of patients with thymomas had pure red cell aplasia. T-cell mediated erythroid rejection is considered the mechanism for the production of pure red cell aplasia in patients with thymomas. This is supported by evidence that a sub group for pure red cell aplasia.”
-
Question 81 of 100
81. Question
1 pointsA patient with ascites is suspected to have secondary hyperaldosteronism. The typical levels of electrolytes in an aliquot specimen of urine would be
Correct
Incorrect
Explanation:
Secondary hyperaldosteronism is characterized by sodium retention, and thus decreased urinary sodium excretion, while potassium secretion is normal to increased.
-
Question 82 of 100
82. Question
1 pointsA 72 year old man presents with lower back pain and difficulty in urination. He has also complaints of bone pain. Initial labs show an increase in serum calcium concentration. The most appropriate test at this time is
Correct
Incorrect
Explanation:
Primary hyperparathyroidism is one of the most common causes of hypercalcemia and should be considered in any individual with an elevated calcium level. A single parathyroid adenoma is the underlying pathology in 85% of cases.
The symptoms have become known as “moans, groans, stones, and bones with psychic overtones”. They include feelings of weakness and fatigue, depression, or aches and painful bones, renal stones, abdominal groans, and psychic moans. With more severe disease, a person may have a loss of appetite, nausea, vomiting, constipation, confusion or impaired thinking and memory, and increased thirst and urination. Patients may have thinning of the bones without symptoms, but with risk of fractures. Elevated parathyroid hormone (PTH) levels in the setting of hypercalcemia establish the diagnosis of hyperparathyroidism -
Question 83 of 100
83. Question
1 pointsLow to normal follicle stimulating hormone level (FSH) is found in patients with which of the following?
Correct
Incorrect
Explanation:
A low caloric intake not only interferes with the nutritional needs of a young and growing organism, but also interferes with the homeostatic mechanisms necessary to maintain functioning. Undernutrition and weight loss associated with anorexia nervosa either reverse pubertal changes and prevent menarche from occurring (primary amenorrhea) or regress the hyperthalamo-pituitary-gonadal axis to prepubertal regulation, resulting in low LH, low FSH and minimal estrogen and progesterone plasma levels (secondary amenorhhea).
-
Question 84 of 100
84. Question
1 pointsA man has a semen analysis done. It shows abnormal motility and shape, and a total sperm count of 950,000 (sperm count should be greater than 2,000,000). What would you advise him about his future fertility?
Correct
Incorrect
Explanation:
Abnormal morphology (shape) and motility can prevent the sperm from reaching the egg. The sperm need motility to be able to swim well and survive for a number of hours n the female reproductive tract. If they do meet, abnormal-looking sperm might be incapable of fertilization. The motility of this man´s sperm i.e. poor and therefore he is unlikely to be able to be fertile. In addition, infertility specialists have stated that 1 million motile sperm is the minimum amount of sperm associated with a reasonable chance of pregnancy success at intrauterine insemination. However in-vitro fertilization IVF can be used to treat Infertility due to his oligospermia. The procedure would involve controlled ovarian hyperstimulation, oocyte retrieval, and fertilization with, sperm, embryo culture, and embryo transfer. Impotence is defined as an inability to achieve and/or maintain an erection.
-
Question 85 of 100
85. Question
1 pointsAll of the following are true regarding diabetic neuropathy EXCEPT
Correct
Incorrect
Explanation:
The more severe and long-standing is the diabetes, the worse the neuropathy. Distal symmetrical diabetic polyneuropathy is the most common because of its refractoriness to therapy. Diabetic third nerve palsy, proximal neuropathies, compression neuropathy (carpal tunnel syndrome), and autonomic neuropathies can also occur in long-standing diabetes. Diabetic proximal motor neuropathy is vascular in origin because the blood vessels that supply the motor nerves become damaged and result in progressive nerve ischemia. The Diabetes Control and Complications Trial Research Group showed that intensive treatment with insulin injections delays onset and slows progression of diabetic neuropathy, retinopathy, and nephropathy. Very little can be done to restore sensation, but foot ulcers.
Osteomyelitis and Charcot joints can be prevented by having a physician check the foot every six months and by keeping the toenails clipped and cleaned. Symptoms of pain caused by neuropathy can sometimes be alleviated by adjusting the doses of amitriptyline, phenytoin, desipramine, and topical capsaicin. Topical capsaicin works by inhibiting substance P in the superficial nerve fibers and diminishing the sensation of tenderness. -
Question 86 of 100
86. Question
1 pointsA 31-year-old female diabetic who receives insulin has been increasingly hungry and has gained weight. Her hyperglycemia has been worsening despite adjustments of her insulin doses. The likeliest cause of her problem is
Correct
Incorrect
Explanation:
Rebound hyperglycemia, also known as the Somogyi phenomenon, typically produces the pattern such as this. Such patients usually respond best to lowering insulin doses, rather than elevating them. Stress would not produce these symptoms, nor would apathetic hyperthyroidism or bulimia.
-
Question 87 of 100
87. Question
1 pointsA 28 year old man discovers a mass in his neck. Physical examination reveals a 2 cm mass in one thyroid lobe that does not concentrate radioisotopes on a thyroid scan. Nodule aspiration reveals small “solid balls” of neoplastic follicular cells. They contain microscopic blood vessels and fibrous stroma in their centers. Most likely associated with the diagnosis of this condition is
Correct
Incorrect
Explanation:
The distinctive cell balls described are broken off papillary clusters, and are considered pathognomic of papillary carcinoma of the thyroid. This is the most common form of thyroid carcinoma. It tends to present in the third to fifth decades of life and shows a modest female predominance. Despite its propensity for local lymphatic intrusion (which may cause multifocality of tumor in the thyroid or cervical lymph node metastases) the tumor generally has an excellent prognosis with 90% of the patients having 20 year survival. It is typically associated with a history of radiation to the neck. Pretibial myxedema is associated with Graves´s disease, the most common form of hyperthyroidism. Weight gain is associated with hypothyroidism. If anything, the patient´s cancer would likely lead to weight loss. Cold intolerance is also associated with hypothyroidism. Heat intolerance is seen with hyperthyroidism, as in patients with Grave´s disease.
-
Question 88 of 100
88. Question
1 pointsAll of the following treatments are recommended for osteoporosis EXCEPT
Correct
Incorrect
Explanation:
A high-fiber diet has not been shown to increase the bone density in osteoporosis. Hormone replacement therapy can slow the progression of osteoporosis even greater than five years postmenopause. Adding progestins can decrease the incidence of endometrial hyperplasia. Slow-release fluoride has been approved by the FDA- alendronate -to inhibit osteoclastic activity and reduce the progression of osteoporosis. Calcium carbonate and vitamin D are both important nutrients for stronger bone formation. Elderly patients should also be exposed to sunlight to get enough vitamin D. Calcitonin works by directly inhibiting osteoclast activity via the calcitonin receptor. It directly induces inhibition of osteoclastic bone resorption by affecting actin cytoskeleton which is needed for the osteoclastic activity.
-
Question 89 of 100
89. Question
1 pointsA 26 year old woman was brought to the emergency room by her family because of an acute psychotic episode. On examination she was found to be obese with red-purple pigmented striae on her trunk. Her blood pressure was elevated to 170/96 and her glucose level was elevated to 210mg/ dl. The appropriate test of choice in order to make the correct diagnosis is
Correct
Incorrect
Explanation:
This patient presents with steroid psychosis secondary to excess cortisol level from Cushing´s syndrome. Hypercortisolism can have features of “buffalo hump,” lipomatosis, hirsutism, ecchymosis, and hypertension. A 24-hour urine cortisol level (B) would be a useful screening tool. An oral glucose tolerance test (A) would tell us nothing except that the patient has diabetes. Cushing´s syndrome can cause diabetes. If one suspects hypertensive encephalopathy exclusively a CT of the brain (C) would be useful to rule out hemorrhage. Lupus patients can also present with cerebral psychosis, but none of the other features would be present. An ANA level (D) would be ordered if one suspects lupus. An elevated aldosterone level (E) is present in Conn´s syndrome. This would present with hypertension, hypernatremia, and hypokalemia. None of the other features would be present.
-
Question 90 of 100
90. Question
1 pointsA 19 year old boy presents with symptoms of color blindness, sexual impotence, and lack of smell sensation. Examination demonstrates cleft lip, eunuchoidal habitus, mild gynecomastia, and anosmia. He has a family history of similar symptoms. The most likely diagnosis is
Correct
Incorrect
Explanation:
Kallmann´s syndrome is familial hypogonadotropic hypogonadism. The hallmark is lack of lutenizing hormone, follicular stimulating hormone, anosmia, and sexual infantilism. The trait is transmitted as an X linked recessive disorder. Anorexia nervosa is a psychologically based eating disorder in patients in whom no organic cause of emaciation can be identified. Noonan´s syndrome or male Turner´s syndrome is associated with testicular dysfunction with impairment of both sperm and androgen production and elevated serum LH and FSH. Polycystic ovary syndrome is characterized by enlarged ovaries and, thus, is only present in women. Males with Kartagener´s syndrome, which is characterized by defective cilia, are infertile because cilia cannot move sperm along the reproductive tract. Infertility does not necessarily affect sexual potency and does not result in the development of secondary female sex characteristics in males.
-
Question 91 of 100
91. Question
1 pointsAll of the following predispose to calcium stone formation, EXCEPT
Correct
Incorrect
Explanation:
Primary hyperparathyroidism results in hypercalciuria, which raises the urine super saturation of calcium phosphate and/or calcium oxalate. Hyperuricosuria probably induces stone formation by heterogeneous nucleation of calcium oxalate by crystals of sodium hydrogen urate or uric acid. Distal renal tubular acidosis prevents the establishment of a normal pH gradient between urine and blood. The resulting alkalotic urine, associated with hypercalciuria and a low urine citrate level, cause a supersaturation with calcium phosphate, predisposing to calcium phosphate stone formation. Fat malabsorption caused by conditions such as jejunalileal bypass, ileal resection, or chronic diseases of the pancreas, results in overabsorption of dietary oxalate. Hyperoxaluria leads to tubulointerstitial nephropathy and stone formation. Idiopathic hypercalciuria is the most frequent abnormality found in patients with a family history of nephrolithiasis and is associated with an increase in intestinal calcium absorption.
-
Question 92 of 100
92. Question
1 pointsA 22 year old female presents with symptoms of heat intolerance, tremors, weight loss, diarrhea, and insomnia. On examination, her eyes seemed to be “bulging” out. Which of the following is NOT a treatment of choice for this condition?
Correct
Incorrect
Explanation:
This patient has thyrotoxicosis; steroids are not the recommended treatment. The most common cause of thyrotoxicosis is Grave´s disease, which this patient exhibits most symptoms of. The exophthalmia, or bulging eyes, are characteristic of these patients. Currently the definitive treatment of choice is iodine 131 radiation treatment to the thyroid gland. Beta blockers are given initially to alleviate the tachycardia and palpitations associated with excess thyroid hormones. Propylthiouracil (PTU) is given prior to radiation treatment to halt the production of T3 and T4 from the thyroid gland. The main side effect of PTU is neutropenia. Tapazole is another antithyroid medication which prevents the conversion of T4 to T3. T3 is the main hormone which works on the end-organ receptors.
-
Question 93 of 100
93. Question
1 pointsA woman aged 77 years presents with weakness, fatigue, anorexia, and symptoms of salt craving. Her blood pressure was 80/60 mmHg on examination and she had hyperpigmentation. Investigations reveal hyperkalemia. The most likely diagnosis is
Correct
Incorrect
Explanation:
Chronic primary adrenocortical insufficiency is associated with hyperkalemia, hyperreninemia, and hypotension. The hyperpigmentation is secondary to excessive ACTH production from the pituitary gland. Hyporeninemic hypoaldosteronism causes hyperchloremic metabolic acidosis. There is mineralocorticoid deficiency and low renin concentrations, as well as severe hyperkalemia. Cushing´s disease is secondary to excess cortisol levels. This can be from an elevation of ACTH from the pituitary gland. Conn´s syndrome is primary hyperaldosteronism. This disease causes hypertension, hypernatremia, and hypokalemia. Secondary adrenal insufficiency is from inadequate stimulation of the adrenal cortex by ACTH. Clinically significant mineralocorticoid deficiency is rare. Hyperkalemia and hyperpigmentation are rare.
-
Question 94 of 100
94. Question
1 pointsA 51 year old woman gradually develops fatigue, depression, and dysphagia. Which one of the following should be included in the initial diagnostic assessment?
Correct
Incorrect
Explanation:
The patient´s symptoms are consistent with primary hypothyroidism due to a goiter. The biofeedback regulation of the thyroid would lead to elevated TSH levels in an attempt by the pituitary to compensate for the lack of thyroxine production by the thyroid. This condition is not associated with any of the other answer choices.
-
Question 95 of 100
95. Question
1 pointsA 49 year old obese woman presents with new onset diabetes. Her physician´s diagnosis is syndrome X. The set of symptoms that physician probably fond in this patient include
Correct
Incorrect
Explanation:
Syndrome X represents a cluster of cardiovascular risk factors, such as glucose intolerance, hypertension, coronary artery disease, dyslipidemia, and obesity. It is mainly associated with insulin resistance. Acromegaly, Cushing´s syndrome, and pheochromocytoma are all associated with production of counterinsulin hormones which causes glucose intolerance. Sedentary lifestyle, smoking, and progressive weight gain are all associated with an increase in cardiovascular risk, but not necessarily insulin resistance. Chronic pancreatitis leads to destruction of beta cells and, thus, less production of insulin. This would lead to glucose intolerance. Microvascular changes such as retinopathy and diabetic foot ulcers, as well as macrovascular changes such as atherosclerosis, are all complications of uncontrolled diabetes.
-
Question 96 of 100
96. Question
1 pointsA woman has weight gain, lethargy, constipation, and amenorrhea. Investigations reveal a decrease in both serum TSH and serum T4 levels. Injection of TRH fails to produce the expected increase in TSH. The most likely cause of the patient´s hypothyroidism is
Correct
Incorrect
Explanation:
A decrease in both serum T4 and TSH could result from either a pituitary defect or a hypothalamic defect. In the case of the hypothalamic defect (tertiary hypothyroidism), decreased secretion of TRH leads to decreased TSH secretion and hence, decreased T4 secretion. In secondary hyperthyroidism, a decrease in TSH secretion due to a pituitary defect (e.g., by tumors, radiation surgery, Sheehan syndrome, empty sella syndrome) is responsible for the decreased T4. The TRH stimulation test can be used to distinguish between these two possibilities. Failure of TSH to increase after injection of TRH indicates a pituitary detect. Hashimoto thyroiditis is an autoimmune disease that leads to primary hypothyroidism (low serum T4) and increased serum TSH. The autoantibodies ultimately lead to thyroid failure and decreased secretion of thyroid hormones. The decrease in negative feedback at the hypothalamus and pituitary leads to increased secretion of TSH.
Iodine deficiency can lead to hypothyroidism due to inadequate iodine for thyroid hormone production. As in primary hypothyroidism, plasma levels of TSH are increased due to loss of negative feedback. A normal to prolonged increase in TSH after injection of TRH indicates a hypothalamic defect or tertiary hypothyroidism. T4 receptor insensitivity also presents with signs and symptoms of hypothyroidism. However, the negative feedback effects of T4 at the hypothalamus and pituitary which are also mediated by thyroid hormone receptors would be reduced and lead to increased TSH and T4 in the blood. -
Question 97 of 100
97. Question
1 pointsA 56 year old type 1 diabetic male presents to the emergency department after experiencing severe diarrhea for the past 5 days. Investigation reveals the following:
Metabolic acidosis with a normal gap
Potassium level of 3.2mEq/L
Urine pH of 5.1
Which would increase the urine pH but worsen the hypokalemia and acidosis?Correct
Incorrect
Explanation:
There are two basic types of acid-base disorders. They are respiratory (affecting the blood pH by causing changes in PCO2) and metabolic (affecting the blood pH by causing changes in HCO3). Acetazolamide is an oral medication that inhibits the enzyme carbonic anhydrase, leading to a reduction of aqueous humor production and a subsequent decrease in intraocular pressure (IOP). This agent is indicated for the prompt lowering of IOP in patients with glaucoma as well as adjunct treatment of edema caused by heart failure and epilepsy. Acetazolamide is contraindicated for use in patients with hypokalemia, hyponatremia, metabolic acidotic states, as well as severe hepatic/renal dysfunction. In contrast to diarrhea, which causes the development of acidic urine, acetazolamide produces alkaline urine. The reason that acetazolamide causes acidosis is that inhibition of carbonic anhydrase is associated with a rapid rise in urinary HCO3 excretion to approximately 35% of filtered load. This along with inhibition of titratable acid and NH4+ secretion in the collecting duct system results in an increase in urinary pH to approximately 8 and development of metabolic acidosis. Acetazolamide causes hypokalemia because the fractional excretion of Na may be only 5% and the fractional excretion of K can be upto 70%. The increased excretion of K+ is, in part, secondary to increased delivery of Na to the distal nephron. Other mechanisms contributing to enhanced K+ excretion include flow-dependent enhancement of K+ secretion by the collecting duct, nonosmotic vasopressin release, and activation of the renin-angiotensin-aldosterone axis. The administration of antacids (large amounts), sodium bicarbonate and diuretics, such as furosemide would lead to the development of metabolic alkalosis or help to correct the patient´s metabolic acidosis. Remember sodium bicarbonate and antacids are basic substances and would be expected to help correct the patient´s metabolic acidotic state. With respect to loop diuretics such as furosemide, these agents increase salt delivery to the collecting duct. Increased delivery leads to increased secretion of K+ and H+ by the duct, causing hypokalemic metabolic alkalosis. This toxicity is a function of the magnitude of the diuresis and can be reversed by K+ replacement and correction of hypovolemia. Respiratory acidosis is often seen in patients with chronic lung diseases, and when opioid analgesics such as hydrocodone, oxycodone, and morphine are administered in large doses. In these patients, the potassium level would not be affected. Theophylline administration is sometimes associated with the development of respiratory alkalosis. Theophylline does not affect a patient´s potassium level.
-
Question 98 of 100
98. Question
1 pointsA type 2 diabetic aged 62 years is started on a new medication. She later presents with malaise, myalgias, respire to distress, and increased somnolence. She has an anion gap of 26, HCO3- of 17 mmol/L, and an arterial blood pH of 7.27. She is most likely receiving which medication?
Correct
Incorrect
Explanation:
Lactic acidosis, characterized by elevated blood lactate, decreased arterial blood pH, decreased bicarbonate, and electrolyte imbalances with an elevated anion gap (normal= 10-12), is a rare but serious complication of metformin administration. The onset of lactic acidosis is usually accompanied by several nonspecific signs and symptoms, including malaise myalgias, respire to distress, and increased somnolence. There may be associated hypothermia, hypotension, and resistant bradyarrhythmias as the condition progresses. When this condition occurs, it is fatal in approximately 50% of cases. Metformin should not be used in patients with renal insufficiency, heart failure, conditions predisposing to hypoxemia, or lactic acidosis. Glucagon is a polypeptide hormone produced by the alpha cells of the islets of Langerhans in the pancreas. It stimulates the conversion of glycogen to glucose in the liver. This hormone is available commercially to be administered in an intramuscular injection for the emergency treatment of severe hypoglycemia in diabetic patients when the administration of oral glucose is not possible. The most common adverse effects include pain at the site of the injection as well as hyperglycemia. Glyburide is a sulfonylurea associated with the development of hypoglycemia and cholestatic jaundice. Miglitol is an alpha-glucosidase inhibitor commonly associated with the development of abdominal discomfort and flatulence. Repaglinide is the non sulfonylurea moiety of glyburide; it is commonly associated with hypoglycemia, nausea, and vomiting. -
Question 99 of 100
99. Question
1 pointsA 36 year old woman has recent fatigue, weight gain, dry skin, hair loss and cold intolerance.. Exam is unremarkable. T4 level is reduced and TSH is elevated. She is taken for surgery where her thyroid gland is found to be uniformly firm and moderately enlarged. Neck lymphadenopathy is also noted. What is likely to be present in her?
Correct
Incorrect
Explanation:
Hashimoto thyroiditis is an autoimmune disorder that often results in hypothyroidism, which is supported by the symptoms of fatigue, weight gain, dry skin, hair loss, and cold intolerance. The thyroid gland is moderately enlarged, and often becomes uniformly firm due to the chronic inflammation. T4 is reduced, and as a result, TSH is elevated due to reduced feedback inhibition. Lymphadenopathy identified in the vicinity is almost pathognomonic for an autoimmune thyroiditis. Patients with Hashimoto thyroiditis may have elevated levels of anti-thyroid peroxidase or antithyroglobulin antibodies. Patients with medullary carcinoma of the thyroid gland will have elevated calcitonin levels due to proliferation of parafollicular C cells. This disease is commonly associated with MEN I and II. However, the presentation of hypothyroidism and a uniformly firm mass is not suggestive of carcinoma. Graves disease is distinguished from other causes of hyperthyroidism by the presence of exophthalmos. It is caused by thyroid-stimulating antibodies resulting in a goiter and hyperthyroidism. Since T4 is reduced and TSH is elevated, the patient certainly does not have this disease. Psammoma bodies are hallmarks of papillarycarcinoma of the thyroid. Once again, the physical features of the thyroid gland in this patient are not suggestive of cancer. Subacute thyroiditis, or de Quervain thyroiditis, is a self-limited hypothyroidism often following a flulike illness. However, the thyroid gland in this situation tends to be very tender and the condition often resolves by itself. -
Question 100 of 100
100. Question
1 pointsA 52 year old non-smoker male was diagnosed with type 2 diabetes 1 year back. His BP is 156/88 mmHg, cholesterol is 5.3 mmol/L (<5.2) and has a BMI of 29. His HbA1c is 7.9% (3.8-6.4), he currently takes only Metformin 500mg bd. The single intervention most likely to reduce his overall risk of Microvascular and macrovascular events is
Correct
Incorrect
Explanation:
The UKPDS showed that effective antihypertensive therapy reduced the risk of cardiovascular events and microvascular complications. Lowering HbA1c only resulted in a significant reduction in microvascular event Lipid lowering therapy benefits patients with diabetes as much as those without diabetes in preventing macrovascular events in sub group analyses but has no effect on microvascular events demonstrated so far Aspirin is recommended to type 2 patients with one other cardiovascular risk factor but there is little trial evidence of efficacy. Weight reduction may reduce progression to overt diabetes from states of impaired glucose tolerance but has not been demonstrated to reduce microvascular risk in diabetes.